Of the votes cast on a certain proposal, 80 more were in

This topic has expert replies
Moderator
Posts: 2244
Joined: Sun Oct 29, 2017 2:08 pm
Followed by:2 members

Timer

00:00

Your Answer

A

B

C

D

E

Global Stats

Princeton Review

Of the votes cast on a certain proposal, 80 more were in favor of the proposal than were against it. If the number of votes against the proposal was 40 percent of the total vote, what was the total number of votes cast? (Each vote cast was either in favor of the proposal or against it.)

A. 480
B. 400
C. 300
D. 240
E. 160

OA B.

Legendary Member
Posts: 2214
Joined: Fri Mar 02, 2018 2:22 pm
Followed by:5 members

by deloitte247 » Sun Nov 11, 2018 12:27 pm
Vote against the proposal = 40% of the Total votes
Vote for the proposal = 100 - 40 = 60% of the Total vote
Total vote is unknown
$$Let\ total\ vote\ =\ x$$
of the total vote x, 80 more were in favor of the proposal than were against it ; thus
$$\left(60\%\ of\ \ x\right)-\left(40\%\ of\ x\right)=80$$
$$\left(\frac{60}{100}\cdot x\right)-\left(\frac{40}{100}-x\right)=80$$
$$0.6x-0.4x=80$$
$$0.2x=80$$
dividing both sides by the coefficient of x
$$\frac{0.2x}{0.2}=\frac{80}{0.2}$$
$$x=400$$
$$answer\ is\ Option\ B$$

GMAT/MBA Expert

User avatar
GMAT Instructor
Posts: 7240
Joined: Sat Apr 25, 2015 10:56 am
Location: Los Angeles, CA
Thanked: 43 times
Followed by:29 members

by Scott@TargetTestPrep » Mon Jan 21, 2019 6:11 pm
AAPL wrote:Princeton Review

Of the votes cast on a certain proposal, 80 more were in favor of the proposal than were against it. If the number of votes against the proposal was 40 percent of the total vote, what was the total number of votes cast? (Each vote cast was either in favor of the proposal or against it.)

A. 480
B. 400
C. 300
D. 240
E. 160
Let x = the total number of votes cast. So 0.4x votes were against the proposal, and 0.6x votes were in favor of the proposal. We can create the equation:

0.6x - 0.4x = 80

0.2x = 80

x = 400

Answer: B

Scott Woodbury-Stewart
Founder and CEO
[email protected]

Image

See why Target Test Prep is rated 5 out of 5 stars on BEAT the GMAT. Read our reviews

ImageImage

GMAT/MBA Expert

User avatar
Elite Legendary Member
Posts: 10392
Joined: Sun Jun 23, 2013 6:38 pm
Location: Palo Alto, CA
Thanked: 2867 times
Followed by:511 members
GMAT Score:800

by [email protected] » Tue Jan 22, 2019 10:54 am
Hi All,

We're told that of the votes cast on a certain proposal, 80 MORE were in favor of the proposal than were against it and the number of votes against the proposal was 40 percent of the TOTAL vote. We're asked for the total number of votes cast? (Each vote cast was either in favor of the proposal or against it). This question can be approached in a number of different ways, including by TESTing THE ANSWERS.

To start, since the difference between the number of "yes" votes and the number of "no" votes is 'round number' (in this case, 80), it's likely that the TOTAL number of votes is ALSO a 'nice' round number. We have two of those in the answer choices (re: 400 and 300). Let's TEST Answer B first.

Answer B: 400 votes
IF... there are 400 votes,
then 40% of the 400 were "no" votes --> (.4)(400) = 160
and the remaining 400 - 160 = 240 were "yes" votes
The difference in "yes" votes and "no" votes is 240 - 160 = 80
This is an exact match for what we were told, so this MUST be the answer.

Final Answer: B

GMAT assassins aren't born, they're made,
Rich
Contact Rich at [email protected]
Image

User avatar
GMAT Instructor
Posts: 1449
Joined: Sat Oct 09, 2010 2:16 pm
Thanked: 59 times
Followed by:33 members

by fskilnik@GMATH » Wed Jan 23, 2019 4:02 am
AAPL wrote:Princeton Review

Of the votes cast on a certain proposal, 80 more were in favor of the proposal than were against it. If the number of votes against the proposal was 40 percent of the total vote, what was the total number of votes cast? (Each vote cast was either in favor of the proposal or against it.)

A. 480
B. 400
C. 300
D. 240
E. 160
$$? = t\left( {{\rm{total}}} \right)\,\,\,\,\left\{ \matrix{
\,{\rm{against}}\,\,\,{\rm{ = }}\,\,a\,\,\mathop = \limits^{{\rm{stem}}} \,\,{2 \over 5}t \hfill \cr
\,{\rm{for}}\,\,{\rm{ = }}\,\,t - a\,\,\mathop = \limits^{{\rm{stem}}} \,\,80 + a\,\,\, \Rightarrow 2a = t - 80 \hfill \cr} \right.\,\,\,\, \Rightarrow \,\,\,2\left( {{2 \over 5}t} \right) = t - 80\,\,\,\, \Rightarrow \,\,\,{1 \over 5}t = 80\,\,\,\, \Rightarrow \,\,\,\left( {\rm{B}} \right)$$

We follow the notations and rationale taught in the GMATH method.

Regards,
Fabio.
Fabio Skilnik :: GMATH method creator ( Math for the GMAT)
English-speakers :: https://www.gmath.net
Portuguese-speakers :: https://www.gmath.com.br

GMAT/MBA Expert

User avatar
GMAT Instructor
Posts: 7240
Joined: Sat Apr 25, 2015 10:56 am
Location: Los Angeles, CA
Thanked: 43 times
Followed by:29 members

by Scott@TargetTestPrep » Sun Jan 27, 2019 6:13 pm
AAPL wrote:Princeton Review

Of the votes cast on a certain proposal, 80 more were in favor of the proposal than were against it. If the number of votes against the proposal was 40 percent of the total vote, what was the total number of votes cast? (Each vote cast was either in favor of the proposal or against it.)

A. 480
B. 400
C. 300
D. 240
E. 160
We can let n = the total number of votes. Thus, 0.4n votes were against the proposal, and 0.6n votes were in favor of the proposal. We can create the equation:

0.6n - 0.4n = 80

0.2n = 80

n = 80/0.2 = 800/2 = 400

Answer: B

Scott Woodbury-Stewart
Founder and CEO
[email protected]

Image

See why Target Test Prep is rated 5 out of 5 stars on BEAT the GMAT. Read our reviews

ImageImage